Chapter 5. Diagnosis of Pain States Flashcards

1
Q
  1. A59-year-old female comes to your office complaining
    of moderately severe low back pain
    and right buttock pain which is exacerbated
    with prolonged sitting. On physical examination
    there is sciatic notch tenderness and the
    pain is exacerbated with flexion, adduction,
    and internal rotation of the right hip. Which of
    the following is the most likely diagnosis?
    (A) L5-S1 facet syndrome
    (B) Piriformis syndrome
    (C) Sacroiliac (SI) joint syndrome
    (D) Sciatica
    (E) L3 radiculopathy
A
  1. (B) The piriformis syndrome was originally
    described by six common characteristics
    (1) trauma; (2) pain in the muscle with sciatica
    and difficulty in walking; (3) worsening with
    squatting or lifting; (4) a sausage-like mass
    within the muscle; (5) positive Lasègue sign;
    and (6) gluteal atrophy. The female to male
    ratio is 6 to 1.
    There are many approaches to evaluate
    piriformis syndrome. One method is in the sitting
    position which involves the examiner
    stretching the piriformis muscle by passively
    moving the hip into internal rotation reproducing
    buttock pain which is relieved by the
    examiner passively moving the hip into external
    rotation. The patient then actively rotates
    the hip against the resistance which reproduces
    buttock pain. Furthermore, there is generally
    point tenderness on palpation of the
    belly of the piriformis muscle. There tends to
    be prolongation of the H-reflex with flexion,
    adduction, and internal rotation.
How well did you know this?
1
Not at all
2
3
4
5
Perfectly
2
Q
  1. A77-year-old female comes to your office complaining
    of 6 months of severe right buttock
    pain radiating into the right lower leg. The pain
    is also present at night and not uncommonly
    interferes with sleep. The pain is severe with
    sitting or lying on her back or right side, however,
    quickly dissipates with normal erect posture.
    Which of the following is the most likely
    diagnosis?
    (A) Snapping bottom
    (B) Sciatica
    (C) Radiculopathy
    (D) Piriformis syndrome
    (E) Weaver’s bottom
A
  1. (E) In classic weaver’s bottom (ischiogluteal
    bursitis)—the patients invariably get pain sitting
    which goes away upon standing or lying
    on their contralateral side. However, the pain
    promptly returns upon resuming a seated position.
    Typically, the patient can consistently
    point to the spot where it hurts with their finger
    and state “it hurts right here.” On physical
    examination, tenderness is evoked with palpation
    over the ischiogluteal bursa.
How well did you know this?
1
Not at all
2
3
4
5
Perfectly
3
Q
  1. A 53-year-old male comes to your office complaining
    of foot pain (predominantly in the
    heel—but also with diffuse plantar symptoms)
    which also occurs at night and can be exacerbated
    by prolonged standing or walking. It is
    associated with weakness of the phalanges
    (impairing the pushing off phase of walking) as
    well as sensory loss and paresthesia. After a
    complete history and physical examination are
    completed, which of the following is the next
    most appropriate step?
    (A) Magnetic resonance imaging (MRI) of
    the ankle
    (B) MRI of the lumbar spine
    (C) Initiate anti-inflammatory medications
    (D) Trial of arch support
    (E) Electrodiagnostic testing
A
  1. (E) Imaging studies are most appropriate with
    bony point tenderness or when the differential
    diagnosis is likely calcaneal stress, fracture
    Paget disease, tumors, calcaneal apophysitis
    (Sever disease in adolescents), or calcaneal
    stress fracture. The most appropriate diagnostic
    evaluation for suspected tarsal tunnel syndrome
    is electrodiagnostic evaluation.
How well did you know this?
1
Not at all
2
3
4
5
Perfectly
4
Q
  1. A 53-year-old male comes to your office complaining
    of foot pain (predominantly in the
    heel—but also with diffuse plantar symptoms)
    which also occurs at night and can be exacerbated
    by prolonged standing or walking. It is
    associated with weakness of the phalanges
    (impairing the pushing off phase of walking) as
    well as sensory loss and paresthesia. Which of
    the following is the most likely diagnosis?
    (A) Morton neuroma
    (B) Peripheral neuropathies
    (C) Medial plantar nerve entrapment
    (D) Tarsal tunnel syndrome
    (E) March fracture
A
  1. (D) The tarsal tunnel located behind and inferior
    to the medial malleolus. It is bounded on
    the lateral aspect by the tibia and medially by
    the flexor retinaculum (laciniate ligament). Its
    contents include the tibial nerve, posterior tibial
    tendons, flexor digitorum longus tendon, flexor
    hallucis longus tendon, tibial artery, and tibial
    vein. Within the tarsal tunnel or immediately
    distal to it, the tibial nerve divides into the
    medial and lateral plantar nerves. The calcaneal
    branch originates variably above or below the
    flexor retinaculum to supply the heel and calcaneal
    skin. The tarsal tunnel syndrome most
    commonly arises from trauma (eg, fractures,
    ankle dislocations) and is characterized by foot
    pain and paresthesia, as well as potentially by
    sensory loss and Tinel sign at the ankle. The
    pain may be similar to carpal tunnel syndrome
    in that it often occurs at night. Furthermore, it
    may be exacerbated by prolonged standing or
    walking. Amarch fracture is a stress fracture of
    the metatarsal bone. The second and third
    metatarsals are the most common sites. Patients
    complain of increased intensity of pain with
    activity or exercise. The pain is localized to the
    site of the fracture.
How well did you know this?
1
Not at all
2
3
4
5
Perfectly
5
Q
  1. A47-year-old female comes to your office complaining
    of an aching forearm with discomfort
    and numbness in the thumb and index finger,
    and weakness in the hand. Apositive Tinel sign
    is present in the forearm. Which of the following
    is the most likely diagnosis?
    (A) Anterior interosseous nerve syndrome
    (B) Posterior interosseous nerve syndrome
    (C) Ulnar nerve entrapment
    (D) Pronator syndrome
    (E) Radial nerve entrapment
A
  1. (D) Pronator syndrome may result from compression
    of the median nerve proximal to the
    branching of the anterior interosseous nerve.
    Patients with pronator syndrome generally
    complain of an aching discomfort of the forearm,
    numbness in the thumb and index finger, and weakness in the hand. On physical examination
    there may be tenderness over the proximal
    part of the pronator teres muscle that is
    exacerbated by pronation of the forearm
    against resistance. Resisted pronation may also
    result in paresthesia in the distribution of the
    median nerve. A positive Tinel sign is often
    present at the proximal edge of the pronator
    muscle. If the entrapment is under the bicipital
    aponeurosis this may result in weakness of the
    pronator muscle and depending on the degree
    of compression, weakness of other muscles (eg,
    long flexor muscles of the fingers and thumb,
    abductor pollicis brevis)
How well did you know this?
1
Not at all
2
3
4
5
Perfectly
6
Q
257. Complex regional pain syndrome type II
(CRPS II) differs from CRPS I because in CRPS II
there is
(A) allodynia
(B) movement disorder
(C) sudomotor and vasomotor changes
(D) evidence of major nerve damage
(E) severe swelling
A
  1. (D) CRPS I and CRPS II are clinically indistinguishable.
    The only difference is that in CRPS II
    there is evidence of major nerve damage.
How well did you know this?
1
Not at all
2
3
4
5
Perfectly
7
Q
258. Which of the following range is the temperature
most appropriate to use as a stimulus
when evaluating warm temperature sensation?
(A) 25°C to 30°C
(B) 30°C to 35°C
(C) 35°C to 40°C
(D) 40°C to 45°C
(E) 45°C to 50°C
A
  1. (D) The temperature range to test warm temperature
    sensation is 40°C to 45°C—usually
    done via a glass or metal tube with hot (40°C-
    45°C) water. Temperatures higher than 45°C
    are generally perceived as painful.
How well did you know this?
1
Not at all
2
3
4
5
Perfectly
8
Q
259. Which of the following range is the temperature
most appropriate to use as a stimulus
when evaluating cold temperature sensation?
(A) −5°C to 0°C
(B) 0°C to 5°C
(C) 5°C to 10°C
(D) 10°C to 15°C
(E) 15°C to 20°C
A
  1. (C) The temperature range to test cold temperature
    sensation is 5°C to 10°C—which may be
    done with a thermophore. Temperatures lower
    than 5°C are generally perceived as painful.
How well did you know this?
1
Not at all
2
3
4
5
Perfectly
9
Q
260. Which of the following may potentially facilitate
or perpetuate myofascial trigger points in
some patients?
(A) Low creatine kinase
(B) Low aldolase
(C) Low cholesterol
(D) Low vitamin D
(E) Low vitamin B12 or folate
A
  1. (E) Low levels of vitamin B12 and/or folate may
    be associated with increased trigger points in
    many patients who suffer from myofascial pain
    syndrome. Multiple coexisting systemic conditions
    may also be associated with myofascial
    pain syndrome and should be investigated
    in patients with severe painful myofascial trigger
    points.
How well did you know this?
1
Not at all
2
3
4
5
Perfectly
10
Q
  1. A 39-year-old male with persistent coughing
    attributed to upper respiratory infection (URI)
    comes to your office complaining of moderate
    anterior chest wall pain—it is only on the left
    side—predominantly over the second and third
    costal cartilages. Bulbous swellings and point
    tenderness are present at these sites. Which of
    the following is the most appropriate diagnosis
    for this patient?
    (A) Intercostal neuralgia
    (B) Tietze syndrome
    (C) Acute myocardial infarction
    (D) Pneumonia
    (E) Pleurisy
A
  1. (B) Tietze syndrome (costochondritis) should
    only be diagnosed after other diagnoses are
    ruled out. It is most frequently unilateral
    involving the second and third costal cartilages
    and is characterized by mild to moderately
    severe anterior chest wall pain. The pain is typically
    localized in the region of the costal cartilages
    but may occasionally radiate to the arm
    and shoulder. Tietze syndrome occurs more
    commonly under the age of 40 years. On
    physical examination, tenderness to palpation
    as well as bulbous swelling over the costochondral
    junctions may be found.
How well did you know this?
1
Not at all
2
3
4
5
Perfectly
11
Q
  1. A 66-year-old woman who did not have a history
    of trauma comes to your office complaining
    of acute, severe, constant medial right knee
    pain for 6 weeks. MRI imaging demonstrated
    extensive narrow edema of the medial femoral
    condyle with significant soft tissue edema
    around the superficial and deep compartment
    of the medial collateral ligament (MCL) but
    without MCL disruption. Which of the following
    is the most likely diagnosis?
    (A) Stress fracture
    (B) MCL tear
    (C) Medial meniscal tear
    (D) Spontaneous osteonecrosis of the
    knee (SONK)
    (E) Medical femoral condyle contusion
A
  1. (D) Spontaneous osteonecrosis of the knee
    (SONK) is an entity whose precise pathogenesis
    remains unclear. The pain may be present at
    rest and is generally well-localized without
    trauma or associated incited event. It is classically
    defined as unilateral and spontaneous
    with predilection for the medial femoral
    condyle. It occurs typically in the elderly population
    (> age 60) and is three times more
    common in women than men. Initial radiographs
    tend to be normal.
How well did you know this?
1
Not at all
2
3
4
5
Perfectly
12
Q
  1. A 49-year-old male comes to your office after
    climbing several mountain passes in the
    Pyrenees on a bicycle with thigh complaints.
    He relates to you that he developed a painful
    sensation on the lateral aspect of his right thigh,
    which lasted for about a week. This was followed
    by numbness and paresthesia in the
    same location. Physical examination revealed
    sensory loss in the same location. Which of the
    following is the most likely diagnosis?
    (A) Tensor fascia lata syndrome
    (B) Meralgia paresthetica
    (C) Iliotibial band syndrome
    (D) Greater trochanteric bursitis
    (E) Lumbar radiculopathy
A
  1. (B) Meralgia paresthetica is a painful mononeuropathy
    of the lateral femoral cutaneous nerve
    (LFCN). Although it may be idiopathic in
    nature it is commonly caused by focal entrapment
    of the LFCN as it passes through the
    inguinal ligament. Although there have been
    numerous reported associated conditions,
    some of these include weight change (eg, obesity,
    pregnancy), possibly external compression
    (eg, seat belts, tight clothing), perioperative
    factors/trauma, retroperitoneal tumors, and
    strenuous walking/cycling (the iliopsoas muscle
    and tensor fascia lata are heavily involved in
    walking and/or cycling movement).
How well did you know this?
1
Not at all
2
3
4
5
Perfectly
13
Q
  1. A43-year-old male runner comes to your office
    complaining of a dull ache in the anterior
    aspect of the lower legs bilaterally which occurs
    about 10 minutes into his running routine each
    time he runs and dissipates with rest. The
    patient states that he needs to stop running
    because of this ache and also notes dysesthesia
    in the first web space of both feet. Which of
    the following is the most likely diagnosis?
    (A) Shin splints
    (B) Stress fractures
    (C) Chronic osteomyelitis
    (D) Periostitis
    (E) Chronic exertional anterior compartment
    syndrome of the lower leg
A
  1. (E) Chronic exertional compartment syndrome
    of the anterior tibial compartment may occur in
    runners, soccer players, and racers and may
    present with a fullness in the anterior compartment,
    exacerbation of pain on passive dorsiflexion
    of the great toe, weakness of the
    extensor hallucis longus muscle, and decreased
    sensation in the first web space. Symptoms are
    usually bilateral 75% to 95% of the time
How well did you know this?
1
Not at all
2
3
4
5
Perfectly
14
Q
  1. A32-year-old construction worker felt a sharp
    pain in his back radiating down to the heel of
    his right foot after lifting a large, metal girder.
    Two days later he noticed numbness in the sole
    of his right foot and fifth toe. Physical examination
    is notable for a decreased ability to walk
    on his toes, a positive straight leg raising test on
    the right, and a markedly diminished ankle
    jerk reflex. Which of the following is the most
    likely diagnosis?
    (A) L4-5 herniated disc
    (B) Discogenic low back pain
    (C) L5-S1 herniated disc
    (D) Spinal stenosis
    (E) Piriformis syndrome
A
  1. (C) Symptoms from an L5-S1 herniated disc
    are typically experienced in the distribution of
    the S1 nerve root. These symptoms may
    include pain or sensory changes in the calf, lateral
    border of the foot, heel, sole, and sometimes
    fourth and fifth toes. On physical
    examination, the patient may have diminished
    strength in the gastrocnemius, soleus, and the
    peroneus longus and brevis muscles. An L4-5
    herniated disc most frequently results in L5 symptoms, which include diminished sensation
    in the lateral leg, dorsum of the foot, and
    the first two toes. Spinal stenosis is narrowing
    of the spinal canal that occurs with aging.
    Patients may present with decreased strength
    and loss of sensation, but with central stenosis
    it is usually nondermatomal. Piriformis syndrome
    is an uncommon cause of buttock pain
    and/or sciatica that is caused by sciatic nerve
    compression by the piriformis muscle. Although
    sciatica is often present, pain from piriformis
    syndrome is nonadicular, and hence straight
    leg raising tests should not be positive.
    Discogenic pain is pain that results from internal
    disc disruption. The neurologic examination
    should be nonfocal when pain results
    solely from internal disc derangement.
How well did you know this?
1
Not at all
2
3
4
5
Perfectly
15
Q
  1. An 80-year-old man presents with a 2-year
    history of low back pain radiating down from
    both legs to his ankles. He also notes numbness
    in his left foot and slight weakness. The pain is
    increased with walking and relieved within
    seconds of cessation of activity. Leaning forward
    eases his pain and lying supine relieves it.
    Which of the following is the most likely
    diagnosis?
    (A) Herniated nucleus pulposus
    (B) Facet arthropathy
    (C) Muscle spasm
    (D) Arachnoiditis
    (E) Spinal stenosis
A
  1. (E) As we age, our spinal canal starts to narrow.
    This narrowing is a result of many different
    processes including disc bulging from a progressive
    loss of disc height and elasticity,
    hypertrophy of the facet joints and ligamentum
    flavum and osteophyte formation.
    Technically, the term “spinal stenosis” can refer
    to central canal stenosis, lateral recess stenosis,
    or foraminal stenosis. The typical presentation
    of someone with spinal stenosis is an elderly
    person with low back and leg pain brought on
    by walking, especially on stairs or hills.
    Frequently, the pain is bilateral. In contrast to
    vascular claudication, patients with neurogenic
    or pseudoclaudication often find that the cessation
    of walking brings immediate pain relief.
    Like spinal stenosis, facet arthropathy is more
    common in the elderly, but the pain does not
    typically radiate into the lower leg and is usually
    not associated with loss of sensation.
How well did you know this?
1
Not at all
2
3
4
5
Perfectly
16
Q
  1. A 31-year-old woman presents to your office
    with marked pain and swelling in her ankle
    6 weeks after an open reduction internal fixation
    with casting. On examination, the ankle is
    warm and erythematous. Lightly touching the
    ankle with a cotton swab evokes severe, lancinating
    pain. You suspect CRPS I. Which of the
    following tests will confirm your diagnosis?
    (A) Lumbar sympathetic block
    (B) Phentolamine infusion test
    (C) Triple phase isotope bone scan
    (D) Erythrocyte sedimentation rate
    (E) None of the above
A
  1. (E) In the early 1990s, a panel of experts
    reached a consensus that the terms “reflex sympathetic
    dystrophy” and “causalgia” had lost
    their utility as clinical diagnoses and suggested
    a new nomenclature be adopted. The new
    terms designated for these conditions are “CRPS
    types I and II”. According to the new diagnostic
    criteria, CRPS need not be maintained by
    sympathetic mechanisms. A three-phase isotope
    bone scan is often positive in CRPS, but a
    normal bone scan does not exclude the diagnosis.
    Erythrocyte sedimentation rate is a nonspecific
    test that is positive in many painful conditions
    including infection, inflammatory arthritides
    and inflammatory myopathies. As a syndrome,
    CRPS is diagnosed by history and physical
    examination. For CRPS I, the diagnostic criteria
    include (1) an initiating noxious event; (2) spontaneous
    pain and/or allodynia occur outside
    the territory of a single peripheral nerve, and
    are disproportionate to the inciting event; (3)
    there is or has been evidence of edema, cutaneous
    perfusion abnormalities, or abnormal
    sudomotor activity, in the region of pain since
    the inciting event; and (4) the diagnosis is
    excluded by the existence of any condition that
    would otherwise account for the degree of pain
    and dysfunction.
How well did you know this?
1
Not at all
2
3
4
5
Perfectly
17
Q
  1. A 46-year-old man complains of worsening
    back and new onset leg pain and paresthesia
    10 weeks after an L4-S1 posterior spinal fusion.
    One week after the surgery, the patient
    reported 85% pain relief. Which radiologic test
    would be most appropriate for detecting the
    cause of failed back surgery syndrome (FBSS)
    in this patient?
    (A) Computed tomographic (CT) scan with
    contrast
    (B) Myelography
    (C) Epidural mapping via the injection of
    contrast under fluoroscopy through a
    catheter inserted through the caudal
    canal
    (D) T2-weighted MRI with contrast
    (E) Further radiologic study is not indicated
    at this point
A
  1. (D) The type and timing of pain after spine surgery
    provide important clues as to the possible
    diagnosis. For example, no change in a patient’s
    pain pattern after surgery may indicate that
    either the wrong surgery was done or the procedure
    was technically unsuccessful. In this
    case, the patient experienced initial pain relief,
    which was followed by worsening back pain
    and new-onset leg pain several weeks later.
    Possible causes of this scenario include epidural
    fibrosis, arachnoiditis, discitis, battered root
    syndrome with perineural scarring, or an early
    recurrent disc herniation. Pseudoarthrosis,
    juxtafusional discogenic pain, and lumbar instability
    can also be causes of FBSS, but in these
    cases the recurrence of pain typically occurs
    much later. For detecting disc pathology, MRI is
    more sensitive than CT or myelography. It is
    also more sensitive than CT for identifying contrast
    enhancement. For the possible etiologies
    that fit this patient’s pain history (ie, arachnoiditis,
    epidural fibrosis, and discitis), contrast
    enhancement with gadolinium will greatly
    enhance the sensitivity of MRI. Epidural mapping
    via the injection of radiopaque contrast
    under fluoroscopy through a catheter inserted
    through the caudal canal is sometimes used to
    determine the location of epidural scar tissue in
    FBSS patients, often as a precursor to epidural
    lysis of adhesions (ie, Racz procedure) or
    epiduroscopy. However, this procedure provides
    very little additional information. In the patient with implanted hardware, foreign ferromagnetic
    metal objects give rise to local distortion
    of the magnetic field, which can greatly
    degrade MRI results. When implants are made
    of non-superparamagnetic materials like titanium,
    MRI distortion is less but the anatomy
    may still be obscured. Since this patient did not
    have hardware implanted, this should not deter
    the use of MRI. Generally, T2-weighted images
    are more sensitive for detecting pathology,
    whereas T1-weighted images are better for discerning
    anatomy. The use of MRI to follow a
    stable, pathologic condition of the lumbar spine
    is controversial. The use of MRI to evaluate a
    patient with chronic low back pain who has
    recently undergone spine surgery and presents
    with new symptoms is justified
How well did you know this?
1
Not at all
2
3
4
5
Perfectly
18
Q
269. Which of the following is false regarding discogenic
low back pain?
(A) Sitting bent forward subjects the
intervertebral disc to a greater amount
of pressure than lying down, standing
or sitting with one’s back straight
(B) It is often diagnosed by using
provocative discography
(C) Because of their caudad position in the
spine, the lower lumbar discs are most
prone to degenerative disc disease (DDD)
(D) Studies have shown a genetic
predisposition to DDD
(E) Intradiscal steroids are an effective
means for treating DDD
A
  1. (E) Sitting bent forward subjects the lumbar
    intervertebral discs to greater stress than standing,
    sitting with one’s back straight, or lying
    down. This helps explain why patients with
    discogenic low back pain often present with
    sitting intolerance. Although controversial,
    discography, with or without CT scanning, is
    still commonly used to diagnose discogenic
    pain. Patients at high risk for false-positive
    discography include those with psychopathology
    and previous back surgery. The lower
    lumbar discs are more likely to develop degenerative
    changes, and hence become pain generators,
    than more cephalad discs because of
    the increased load they bear. Recent studies
    have shown a genetic predisposition for both
    degenerative disc disease and sciatica. Several
    prospective studies have been conducted evaluating
    intradiscal steroids in patients with
    discogenic low back pain, and none have found
    them to be efficacious.
How well did you know this?
1
Not at all
2
3
4
5
Perfectly
19
Q
  1. Which of the following statements concerning
    central pain is true?
    (A) Spinal cord injury is the leading cause of
    central pain in the United States
    (B) Lesions involving spinothalamocortical
    pathways are necessary and sufficient to
    cause central pain
    (C) Central pain is a common sequelae
    following neurosurgic procedures
    (D) Motor cortex stimulation is an effective
    means to treat central pain
    (E) The most typical presentation of central
    pain is a spontaneous, burning sensation
    on the entire body contralateral to
    the lesion site
A
  1. (D) Owing to its high incidence, stroke is the
    leading cause of central pain in the industrialized
    world. The chance of developing central
    pain following spinal cord injury is higher than
    after stroke (30%-50% vs 8%), but the overall
    number of stroke patients with central pain is
    higher. Syringomyelia is the disorder with the
    highest incidence of central pain (60%-80%).
    According to neurosurgical studies conducted
    by V. Cassinari and C.A. Pagni in the 1960s,
    injury to spinothalamocortical pathways is necessary
    but not sufficient to cause central pain.
    The reason why some patients develop central
    pain but others with identical injuries do not is
    unknown. Central pain may occur after neurosurgical
    procedures and intracranial bleeds,
    but these are unusual occurrences. There are
    now several prospective studies showing
    motor cortex stimulation to be an effective
    treatment for central pain. There is no typical
    presentation for central pain. While spontaneous
    pain is almost universal, allodynia also
    affects a majority of central pain patients. The
    time lag between the injury and onset of pain,
    and the location of central pain are extremely
    variable.
How well did you know this?
1
Not at all
2
3
4
5
Perfectly
20
Q
  1. Which of the following is not commonly used
    to diagnose the level of nerve root involvement
    in radicular pain?
    (A) MRI
    (B) CT scan
    (C) Selective nerve root block
    (D) Electromyography (EMG)/nerve
    conduction studies (NCS)
    (E) Epidural injections with local anesthetic
    and steroids
A
  1. (E) MRI is usually the first test used to evaluate
    new-onset radicular pain. CT scan is less sensitive
    than MRI for detecting disc pathology, but
    is used in patients with pacemakers, spinal
    hardware (owing to the poor resolution of MRI
    in patients with ferromagnetic metal objects)
    and when MRI is not available. Selective nerve
    blocks are sometimes used to diagnose nerve
    root pathology prior to surgery, but there is little
    evidence as to whether or not this improves
    outcomes. Although the terms are sometimes
    used interchangeably, selective nerve blocks are
    not the same as transforaminal epidural injections.
    Since transforaminal epidural injections
    typically result in injectate spread to contiguous
    spinal levels, they cannot be considered diagnostic.
    In addition to providing information
    about the site of nerve root lesions, EMG/NCS
    can help determine whether or not the lesion is
    axonal or demyelinating; whether it is focal,
    multifocal or diffuse; and the age, severity, and
    prognosis of the lesion. QST is a subjective test
    used to evaluate large and small fiber neuronal
    dysfunction. It may be helpful in clarifying
    mechanisms of pain, diagnoses, and guiding
    treatment. It is not used to diagnose nerve root
    pathology.
How well did you know this?
1
Not at all
2
3
4
5
Perfectly
21
Q
272. Which of the following conditions is not generally
associated with a painful neuropathy?
(A) Chronic renal failure
(B) Celiac disease
(C) AIDS
(D) Fabry disease
(E) Amyloidosis
A
  1. (A) Chronic renal failure is associated with
    large, myelinated fiber loss that is rarely
    painful. Celiac disease is a chronic inflammatory
    enteropathy resulting from sensitivity to
    gluten. Neurologic complications are estimated to occur in approximately 10% of patients with
    peripheral neuropathy and ataxia being the
    most common. The neuropathy is usually sensory,
    although infrequently motor weakness
    may develop. There is some evidence that the
    neurologic symptoms associated with celiac
    disease may be ameliorated by a gluten-free
    diet. Peripheral neuropathies are reported to
    affect up to 35% of AIDS patients, being more
    common in later stages of the disease. The most
    common neuropathy in AIDS patients is a
    distal sensory polyneuropathy caused by the
    human immunodeficiency virus (HIV). Other
    causes of neuropathy in AIDS patients include
    toxic neuropathies from medications, co-infection
    with cytomegalovirus (CMV) and other organisms,
    and vitamin B12 deficiency. Fabry disease
    is an X-linked, lysosomal storage disease that
    involves the accumulation of galactosylglucosylceramide
    because of deficiency of α-galactosidase
    A. It usually presents in adulthood; if
    symptoms occur in childhood they usually take
    the form of a painful neuropathy. Amyloidosis
    may result in a painful peripheral or autonomic
    neuropathy secondary to deposition of amyloid
    in nervous tissue. In one study, 35% of
    patients with amyloidosis were found to have
    peripheral neuropathy.
How well did you know this?
1
Not at all
2
3
4
5
Perfectly
22
Q
  1. Apreviously healthy 31-year-old woman presents
    to her internist with generalized muscle
    pain, most prominent in her right thigh. The pain
    travels down the back of her leg to the bottom
    of her foot. She also notes progressive numbness
    and weakness in her arms and legs.
    Walking is difficult and a loss of fine motor
    control makes routine tasks like eating a challenge.
    A review of her medical record reveals
    an URI 3 weeks earlier. Which of the following
    is the most likely diagnosis?
    (A) Multiple sclerosis
    (B) Guillain-Barré syndrome
    (C) Chronic fatigue syndrome
    (D) Acute lumbar and cervical
    radiculopathies
    (E) Diabetic neuropathy
A
  1. (B) The patient’s symptoms are most consistent
    with Guillain-Barré (GB) acute inflammatory
    demyelinating polyneuropathy. Patients
    with GB syndrome generally present with diffuse
    muscular or radicular pain followed by
    sensorimotor dysfunction. Most, but not all
    (72%) patients with GB syndrome experience
    pain during the course of their illness. GB syndrome
    affects 1 to 1.5 people per 100,000 and
    shows no age or gender preference. About 60%
    to 70% of cases are preceded by an URI or gastrointestinal
    (GI) illness 1 to 3 weeks before
    symptoms begin. Cerebrospinal fluid (CSF)
    analysis reveals normal pressures, increased
    protein and no cells. The pathology of GB syndrome
    is demyelination, with most patients fully
    recovering. Multiple sclerosis is a demyelinating
    disease that typically presents in early adult
    life. The most common presenting symptom
    of multiple sclerosis is ocular complaints,
    which affects most patients at some time during
    the course of their illness. Spinal cord lesions
    can produce a myriad of sensorimotor problems
    including weakness, spasticity, hyperreflexia,
    bladder dysfunction, sensory loss, and
    diminished temperature sensation and proprioception.
    Central dysesthetic pain affects
    approximately 20% of multiple sclerosis
    patients. The diagnosis of multiple sclerosis is
    usually supported by MRI, with or without
    CSF analysis. Although muscle pain and weakness
    may be present in chronic fatigue syndrome
    (CFS), the hallmark of this disorder is
    disabling physical and mental fatigue present
    for more than 6 months. There is no firm data
    causally linking viral infection to CFS despite
    frequent reported associations. The most
    common presentation of acute radiculopathy is
    pain or sensory changes in a lower extremity.
    The most common form of diabetic neuropathy
    is distal, symmetrical polyneuropathy. It is predominantly
    a sensory disturbance, occurring
    in a stocking-glove distribution. Because the
    feet are innervated by the longest nerves in the
    body, they are usually the first part of the body
    to be affected. Other types of neuropathy that
    may be present in diabetics include lower
    extremity proximal motor neuropathy, truncal
    neuropathy, cranial mononeuropathy, and autonomic
    neuropathy. The cause of diabetic neuropathy
    is most likely related to metabolic and
    ischemic nerve injury
How well did you know this?
1
Not at all
2
3
4
5
Perfectly
23
Q
  1. Which of the following statements is true
    regarding SI joint pain?
    (A) The SI joint is a diarthrodial synovial
    joint designed primarily for stability
    (B) Patrick’s and Gaenslen’s tests are definitive
    diagnostic tests for SI joint pain
    (C) CT scanning is the most sensitive means
    for diagnosing SI joint pain
    (D) Lifting heavy objects is the one of the
    most common causes of SI joint injury
    (E) When diagnostic blocks fail, surgery can
    usually provide long-term pain relief
A
  1. (A) The SI joints are large, paired, diarthrodial
    synovial joints whose primary functions are
    stability and dissipating truncal loads. The
    joints are also involved in limiting x-axis rotation
    and in women, parturition. There are literally
    dozens of provocative tests that have
    been advocated as screening tools for SI joint
    pain, but several studies have shown that these
    tests lack both specificity and high sensitivity.
    On a similar note, CT scanning may show SI
    joint pathology in over 30% of asymptomatic
    control patients, and be negative in over 40% of
    patients with SI pain. The most reliable method
    for diagnosing SI joint pain is through diagnostic
    local anesthetic blocks. The mechanism
    of SI joint injury has been described as a combination
    of axial loading and sudden rotation.
    Common causes of SI joint pain include motor vehicle accidents, falls, athletic injuries, spondyloarthropathies,
    and pregnancy. SI joint injections
    with corticosteroids have been shown in
    some but not all studies to provide short-term
    pain relief. SI joint pain is usually not amenable
    to surgical correction.
How well did you know this?
1
Not at all
2
3
4
5
Perfectly
24
Q
275. Which of the following statements regarding
headaches is false?
(A) The International Headache Society’s
diagnostic criteria for cervicogenic
headaches includes unilaterality of
symptoms and relief of pain by
diagnostic anesthetic blocks
(B) Migraine with aura is more common
than migraine without aura
(C) In chronic tension-type headache, the
average headache frequency is equal to
or greater than 15 days per month
(D) Cluster headaches are more prevalent in
men than in women
(E) Tricyclic antidepressants are a mainstay
of treatment for both migraine and
tension-type headaches
A
  1. (B) In population-based studies, migraine without
    aura is about twice as frequent as migraine
    with aura. Major criteria for the diagnosis of
    cervicogenic headache include signs and symptoms
    of neck involvement such as the precipitation
    of head pain by neck movement or
    external pressure over the upper cervical or
    occipital region, restricted range of motion in
    the neck, unilaterality of head pain with or
    without shoulder or arm pain, and confirmatory
    evidence by diagnostic anesthetic blocks.
    Chronic tension-type headache differs from
    episodic tension-type headache in that the
    average headache frequency is equal to or
    greater than 15 days per month or 180 days
    per year. A shift from peripheral to central
    mechanisms is believed to play a role in the
    evolution of episodic to chronic tension-type
    headache. Cluster headaches typically present
    as a series of intense unilateral headaches
    occurring over a period of 2 weeks to 3 months.
    They are associated with unilateral autonomic
    features such as nasal congestion, rhinorrhea,
    miosis, or lacrimation. The attacks are usually
    brief, lasting between 15 and 180 minutes, and
    occur in the orbital, supraorbital and/or temporal
    regions. Unlike migraine headaches,
    tension-type headaches, temporal arteritis, and
    cervicogenic headaches, cluster headaches are
    more frequent in men, with an average male to
    female ratio of 5 to 1. Tricyclic antidepressants
    have been shown in numerous clinical trials to
    be effective in the prevention of both migraine
    and tension-type headaches.
How well did you know this?
1
Not at all
2
3
4
5
Perfectly
25
276. Which of the following statements regarding postamputation pain is correct? (A) Vascular conditions are the leading cause of both lower and upper extremity amputations (B) There is no relationship between persistent stump pain and phantom limb pain in amputees (C) The intensity of pain and the length of the phantom increases with time (D) Phantom breast pain is a common cause of postmastectomy pain (E) Phantom pain was first described in the American Civil War
276. (D) Phantom breast pain occurs in roughly 20% of mastectomy patients, and phantom sensations in close to half. Originally thought to be rare, phantom limb pain is now recognized to occur in between 60% and 80% of limb amputees. Phantom limb pain must be distinguished from phantom sensations, which occur in over 90% of patients. Vascular conditions account for over 80% of limb amputations in the United States. However, trauma is responsible for approximately 75% of upper extremity amputations. Most researchers have found a statistically significant association between phantom limb pain and persistent stump pain. Although earlier studies found a correlation between preamputation pain and phantom limb pain, more recent studies have not confirmed this relationship. It is widely held that phantom pain diminishes with time and eventually fades away. Though described, phantom pain associated with congenital absence of a limb is rare. Phantom pain is generally worse in the distal part of a limb. Most phantoms shrink with time, with the most distal aspect of a limb being the last to disappear. This is known as “telescoping,” and occurs in approximately half of all limb amputees. Archaeological records demonstrate that purposeful amputations have been practiced since Neolithic times. The concept of “phantom pain” has been recognized for hundreds, if not thousands, of years. In the 16th century, the French military surgeon Ambrose Paré outlined clear distinctions between phantom limb pain, phantom sensation, and stump pain. The term “phantom pain” was coined by Weir Mitchell in the American Civil War. A few years earlier, Mitchell used the word “causalgia” to describe the characteristic autonomic changes found in the extremities of soldiers who suffered major nerve damage.
26
277. Which of the following statements regarding the assessment of pain in pediatric patients is true? (A) Palmar sweating and reduced transcutaneous oxygen concentrations are indicative of pain (B) In a hospitalized 2-year-old child, crying and increased vitals signs are likely to indicate chronic pain (C) The FACES scale and Charleston Pain Pictures provide accurate assessments of pain in preschool children (D) The COMFORT scale and facial action coding system (FACS) are pain instruments used in young children that are based predominantly on facial actions (E) Visual analogue and numerical rating scales are inappropriate pain indices for most adolescents
277. (A) Palmar sweating and reduced transcutaneous oxygen concentration are indicative of, though not specific for, acute pain. In a young child, crying and increased vital signs (eg. heart rate, respiratory rate, and blood pressure) are associated with distress, which includes but is not limited to pain. Other factors that may cause these signs include separation anxiety, hunger, and fear. Unlike acute pain, chronic pain is usually not associated with elevated vital signs. The FACES scale and Charleston Pain Pictures are designed to provide assessments of pain in school aged, not preschool children. The FACS and COMFORT scale are used to assess pain in infants and young children. The FACS is a comprehensive coding system based on a wide range of facial actions. The COMFORT scale is an eight-item scale designed to measure distress (including pain) that includes alertness, calmness, respiratory response, physical movement, blood pressure, muscle tone, and facial tension. Pain scales used in adults such as verbal pain scores, numerical rating scales, and visual analogue scales provide accurate assessments of pain in most adolescents.
27
278. Which of the following statements is not correct regarding herpes varicella zoster? (A) The most common presentation of acute herpes zoster (AHZ) is pain and a vesicular rash in the midthoracic dermatomes (B) The polymerase chain reaction (PCR) is the most common means to diagnose AHZ (C) The incidence of both AHZ and postherpetic neuralgia increases with age (D) There is no generally accepted time period from the onset of AHZ to when a diagnosis of postherpetic neuralgia is made (E) AHZ involving the lumbosacral dermatomes may be misdiagnosed as a herniated disc
278. (B) The most common way to diagnose AHZ is clinically. In a small percentage of patients, AHZ may occur without a rash, a condition known as “zoster sine herpete” (zoster without rash). The PCR is often used to aid in the diagnosis of this condition. In descending order, the most common sites for AHZ are the midthoracic dermatomes, the ophthalmic division of the trigeminal nerve, and the cervical region. The incidence of both AHZ and postherpetic neuralgia increases with age. Other risk factors for AHZ include HIV infection and transplant surgery, which is likely because of the resultant immunosuppression. There is no standard time period after which persistent pain from AHZ is diagnoses as postherpetic neuralgia. Postherpetic neuralgia has been variably defined as the persistence of sensory symptoms 1 month, 6 weeks, 2 months, 3 months, and 6 months after herpes zoster. AHZ affects the lumbosacral dermatomes in between 5% and 15% of patients. Lumbosacral AHZ may be misdiagnosed as a herniated disc
28
279. Which of the following statements regarding electrophysiologic testing is true? (A) Nerve conduction velocities are more likely to decrease in conditions such as alcoholic and diabetic neuropathy that are characterized by Wallerian degeneration than in demyelinating neuropathies such as Guillain-Barré (B) EMG can provide information about the type, extent and timing of injuries to motor units and individual muscle fibers (C) The H reflex can aid in the evaluation of brachial plexus injuries (D) The F response is used to diagnose pure sensory neuropathies (E) EMG can readily identify processes causing muscle denervation (neuropathies), but is incapable of identifying myopathies
279. (B) EMG provides a wealth of information about the integrity, function, and innervation of motor units and (using special techniques) individual muscle fibers. Serial EMG examinations permit monitoring of recovery or disease progression. A normal EMG indicates the absence of motor unit involvement. In neuropathies characterized by Wallerian degeneration, nerve conduction velocities range from low normal to mildly slow. In contrast, demyelinating neuropathies of the acute and chronic inflammatory types produce segmental demyelination, which markedly slows conduction velocities. The H wave is the electrical representation of the tendon reflex circuit. In adults, it is only obtainable in the lower reflexes. It is most prominent during stimulation of the tibial nerve, being particularly helpful in the diagnosis of S1 radiculopathy and predominantly sensory polyneuropathies. The F wave is a late response that is evoked by supramaximal stimulation of a motor nerve. It occurs when a small percentage of the stimulated motor neurons “rebound.” The initial response to stimulation of a motor nerve is the M wave. Unlike H waves, F waves are not true reflexes.
29
280. Which of the following statements is true about quantitative sensory testing (QST)? (A) QST can be used to pinpoint which nerve is injured and where along its path the lesion lies (B) Thermal sensation is used to measure the integrity of large, myelinated nerve fibers (C) A beta function can be evaluated using either a tuning fork or von Frey hair (D) QST can be used to evaluate the function of all different types of nerve fibers (E) An advantage of QST is that it can accurately assess function in uncooperative or incapacitated patients
280. (C) Large, myelinated nerves are more vulnerable to injury than small neurons. The function of large, myelinated A-β function can be measured using both vibratory thresholds and von Frey filaments. QST is used to evaluate the function of individual nerve fibers. It is not useful in determining which nerve is injured and where along its path the injury lies. Both cold and hot thermal sensations are used to measure the function of small myelinated (A-δ) and unmyelinated C fibers. QST cannot be used to assess B (preganglionic autonomic) and A-γ (muscle spindle efferent) function. Adownside of QST is that its accuracy is dependent on the cooperation and reliability of the patient.
30
281. A 38-year-old construction worker presents to you with complaints of right lower extremity pain for the last 8 months. Pain radiates from the lower back to the outer aspect of the right leg and goes down to the dorsum of the right foot. The patient reports a problem with walking and on examination reveals an antalgic gait and inability to do heel-walking on right, though toe-walking is not affected. Strength is 5/5 in all muscle groups except dorsi-flexion of the right ankle which is 4/5 and strength testing for extensor hallucis longus reveals 4/5 strength. Deep tendon reflexes are 2+ at both knees and both ankles. Sensory testing reveals mildly reduced sensation to light touch and pinprick on the dorsum of the right foot when compared to the left foot. This patient most likely has (A) right piriformis syndrome (B) right L4 radiculopathy (C) right L5 radiculopathy (D) right S1 radiculopathy (E) facet arthritis
281. (C) Lumbar radiculopathy most often results from disc herniation. Depending on the level of herniated discs radiculopathy may affect specific nerve roots. Disc herniation at L4-5 and L5-S1 is most likely caused by mobility of the segment. A herniated disc may compromise the nerve root at the same level if displaced laterally in the recess or in the foramen (L4-5 disc affecting L4 nerve root), or it may effect the traversing nerve root to the level below (L4-5 disc affecting L5 nerve root). L5 radiculopathy results in pain, sensory, and motor changes in L5 dermatomal distribution. Pain is usually described as shooting or occasionally aching and burning sensation on the outside of leg radiating to the dorsum of foot. Sensory testing may also reveal a decrease in light-touch and pinprick sensation in the same distribution. L5 radiculopathy also may result in weakness in the extensor hallucis longus and thus heel walking. Deep tendon reflexes may be spared in the lower extremity.
31
282. A46-year-old female with past medical history of depression, anxiety, irritable bowl syndrome, and asthma is referred to you for evaluation of her lower back pain. History reveals onset of generalized pain that started after she was involved in a car accident 4 years ago. Physical examination reveals nonfocal neurologic examination. Musculoskeletal examination reveals multiple areas of hypersensitivity. The patient reports marked pain with moderate digital pressure at base of skull, her neck, front of her chest, her elbows as well as her lower back, and bilateral lower extremities. The patients MRI scan of the lumbar spine reveals preserved disc height, no facet arthritis and minimal disc bulge at L4-5 without any spinal or foraminal stenosis. This patient most likely has (A) fibromyalgia syndrome (B) discogenic pain (C) myofascial pain disorder (D) somatoform disorder (E) opioid hyperalgesia
282. (A) Fibromyalgia syndrome is a common pain condition, estimated to occur in 2.4% of the general population. The syndrome is characterized by widespread musculoskeletal pain, sleep disturbance, psychologic distress, and comorbidity with other pain syndromes [eg, irritable bowel syndrome (IBS), interstitial cystitis, and the female urethral syndrome], which have considerable impact on the everyday life of patients. Fibromyalgia syndrome occurs predominantly in women and demonstrates familial aggregation. Since 1990, the diagnosis of fibromyalgia syndrome has been based on criteria of the American College of Rheumatology (ACR). A key dimension of the ACR criteria is the concept of tender points, 18 specific points on the body surface at which digital palpation elicits pain (11/18 “positive” tender points fulfills an fibromyalgia criteria). It is not uncommon for patients to have other pain pathologies in addition to fibromyalgia. However a complete clinical picture should be viewed before consideration of treatment options especially if it involves interventional procedures. Patient describes above most likely has fibromyalgia as evidenced by the presence of tender points. A negative physical examination except for tender points and hypersensitivity argues against other listed options.
32
283. A25-year-old, healthy female volleyball player has developed severe pain in right hand. This pain started while playing volleyball and after a reported wrist sprain. One month after the initial injury and despite conservative care with nonsteroidal anti-inflammatory drugs (NSAIDs), muscle relaxants, and hand splint to avoid any movement related pain, the patient complains of even worse burning pain. Pain is worse with light touch, even blowing air or rubbing of clothes trigger unbearable pain. The patient also reports her right hand to be cold and often wet because of localized sweating. On examination the patient has a markedly swollen, redappearing hand. Patient is unable to make a fist with her fingers and measurement of temperature reveals a 7°C lower temperature compared to opposite extremity. Which of the following is the most likely diagnosis? (A) CRPS I (RSD) (B) CRPS II (causalgia) (C) Peripheral vascular disease (D) Deep venous thrombosis of upper extremity (E) Median neuralgia
283. (A) Following is the diagnostic criteria for CRPS I: 1. The presence of an initiating noxious event or a cause of immobilization. 2. Continuing pain, allodynia, or hyperalgesia with which the pain is disproportionate to any inciting event. 3. Evidence at some time of edema, changes in skin blood flow, or abnormal sudomotor activity in the region of the pain. 4. This diagnosis is excluded by the existence of condition that otherwise would account for the degree of pain and dysfunction. The patient in the question meets all the criteria for diagnosis of CRPS I (RSD). CRPS II (causalgia) by definition has a known injury to a major nerve. Vascular etiology though possible after trauma, is unlikely to give symptoms of allodynia as well as sudomotor changes. Median neuralgia would result in a similar clinical pain picture but only hand discomfort would be expected to be confined only to the distribution of median nerve.
33
284. A 38-year-old man developed complete T4 spinal cord injury after a motorcycle accident. Two months after the injury the patient continues to complain of severe radiating pain to the front of chest just above nipple line. The pain is worse with light touching and improves with movement restriction and use of morphine on as needed basis. This patient most likely has (A) central dysesthesia syndrome (B) syringomyelia (C) transitional zone pain (D) myofascial pain (E) autonomic dysreflexia
284. (C) Spinal cord injury may result in various types of pain. To provide the most effective treatment—understanding the mechanism of pain is very important. Taxonomy of spinal cord injury pain may be divided into neuropathic or nociceptive pain. The patient in question appears to have most likely nerve root impingement at T4-5, level of his spinal cord injury, resulting in severe T4 neuralgic pain radiating towards the front of chest wall.
34
285. After a car accident 5 days ago, a 42-year-old engineer reports severe neck and midback pain. The patient was rear ended while stopped at a traffic light by a pickup truck. The patient reports severe pain in neck that radiates down to both shoulders and upper arm as well as to the midback region. The pain is a severe stabbing and aching sensation that is markedly exaggerated by movement of neck. Examination reveals otherwise intact neurologic system, 5/5 strength, and intact deep tendon reflexes without any sensory deficit. Imaging studies are essentially normal except for straightening of cervical lordosis. The patient most likely has (A) bilateral C5 radiculopathy (B) myofascial pain (C) fibromyalgia (D) thoracic outlet syndrome (E) malingering
285. (B) Myofascial pain may result after a sudden acceleration-deceleration insult. Neck muscles may reflexly go into spasm. It may also result in straightening of cervical lordosis secondary to spasm of posterior supporting neck muscles. Myofascial pain from cervical neck muscle may radiate between shoulder blades as well into the upper extremity. Negative imaging studies are essential to rule out traumatic disc herniation or fracture. Treatment includes nonsteroidals, muscle relaxants, and physical therapy. In a small percentage of patients, if pain doesn’t resolve trigger point injections or cervical medial branch blocks may provide help with continuing physical therapy.
35
286. A 64-year-old female with a history of coronary artery disease, peripheral vascular disease, and type 1 diabetes mellitus, controlled with insulin, presents to your pain clinic with gradually worsening bilateral leg and feet pain. The patient reports a history of a fall approximately 5 years ago which resulted in severe back and leg pain. That pain resolved, however, the patient started developing numbness and tingling in both legs and feet. On examination the patient reveals otherwise normal appearing legs and feet, patient does have a nonhealing ulcer on her right great toe. Neurologic testing reveals bilateral 5/5 muscle strength and 2+ patellar and ankle reflexes. Sensory testing reveals intact proprioception but reduced sensation to light touch and pinprick. The patient also reported marked sensitivity to light touch. This patient most likely has (A) CRPS I (B) peripheral vascular disease (C) diabetic polyneuropathy (D) lumbar spondylosis (E) central pain
286. (C) In type 1 diabetes mellitus, distal polyneuropathy typically occurs after many years of chronic prolonged hyperglycemia. Conversely, in type 2, it may present after only a few years of poor glycemic control. Occasionally, in type 2, diabetic neuropathy is found at the time of diagnosis (or even predating diagnosis). Diabetic neuropathy can manifest with a wide variety of sensory, motor, and autonomic symptoms. Sensory symptoms may be negative or positive, diffuse or focal. Negative sensory symptoms include numbness; “deadness”; feeling of wearing gloves or walking on stilts; loss of balance, especially with the eyes closed; and painless injuries. Positive symptoms include burning, pricking pain, electric shocklike feelings, tightness, and hypersensitivity to touch. Motor symptoms can cause distal, proximal, or focal weakness. Autonomic symptoms may be sudomotor, pupillary, cardiovascular, urinary, GI, and sexual. A generally accepted classification of diabetic neuropathies divides them broadly into symmetric and asymmetric neuropathies. Symmetric polyneuropathies involve multiple nerves diffusely and symmetrically and are the most common form. The patient in question appears to have symmetrical small and large fiber neuropathy resulting in pain in both legs and feet, and decreased light-touch sensation as well as allodynia.
36
287. A32-year-old female develops severe stabbing, “like an ice pick,” pain at the base of tongue after an infratemporal neurosurgic procedure. Pain comes in paroxysms and last a few seconds and is triggered by swallowing, yawning, and coughing. This patient most likely has (A) trigeminal neuralgia (B) geniculate neuralgia (C) glossopharyngeal neuralgia (D) migraine with atypical aura (E) cluster headache
287. (C) Glossopharyngeal neuralgia is a disorder characterized by intense pain in the tonsils, middle ear, and back of the tongue. The pain can be intermittent or relatively persistent. Swallowing, chewing, talking, sneezing, or eating spicy foods may trigger the disorder. It is often the result of compression of the 9th nerve (glossopharyngeal) or 10th nerve (vagus), but in some cases, no cause is evident. Skull base surgery or surgeries in the infratemporal region may result in damage or irritation of glossopharyngeal nerve. Conservative treatment includes using anticonvulsants. In refractory cases glossopharyngeal nerve block may be helpful. Radiofrequency lesioning or neurolytic treatment should be reserved for resistant cases or ones associated with head and neck cancer. Surgical decompression should be reserved for nonresponders and resistant cases.
37
288. A38-year-old patient care technician while lifting a 400 lb patient heard a pop in his back. The patient developed severe back pain with radiation to the right leg. Patient described the pain as stabbing back pain with electrical sensations down the back of the right leg all the way to the sole of the right foot. On examination the patient appeared very uncomfortable, sitting in a wheel chair. Straight leg raise and cross straight leg raise test was positive. Muscle strength was 5/5 in all muscle groups except plantar flexion at right ankle which was 4/5. Deep tendon reflexes were intact at the patella bilaterally; however, the reflex at the right ankle is diminished compared to the left ankle. The patient most likely has a herniated disc at (A) L4-L5 resulting in L4 nerve root compression (B) L4-L5 resulting in L5 nerve root compression (C) L5-S1 resulting in L5 nerve root compression (D) L5-S1 resulting in S1 nerve root compression (E) L1-L2 resulting in compression of cauda equina
288. (D) Lumbar radiculopathy most often results from disc herniation. Depending on the level and “direction” of herniated discs a resultant radiculopathy may affect specific nerve roots. Disc herniation at L5-S1 is most likely a result of mobility of the segment. A herniated disc may compromise the nerve root at the same level if displaced laterally in the recess or in the foramen (L5-S1 disc affecting L5 nerve root), or it may affect the traversing nerve root to the level below (L5-S1 disc affecting S1 nerve root). S1 radiculopathy results in pain, sensory, and motor changes in S1 dermatomal distribution. Pain is usually described as shooting or occasionally as an aching and burning sensation on the back of thigh radiating to the plantar aspect (sole) of foot. Sensory testing may also reveal a decrease in light-touch and pinprick sensation in the same distribution. S1 radiculopathy also may result in weakness in Plantar flexion and thus toe walking. Most often with significant S1 nerve root compression, ankle reflex is diminished. Examination also may reveal positive straight leg raise and cross straight leg raise test (reproduction of radiating pain in lower extremity by raising the opposite extremity).
38
289. A48-year-old patient after a gunshot wound to the upper chest develops a partial cord transection involving the right spinothalamic tract at T2 level. This patient is most likely to develop loss of pain and temperature sensation: (A) At the level of the transection (B) Below and on right side from the level of transection (C) Below and on left side from the level of transection (D) Patient is not likely to develop central dysesthetic pain (E) Below and bilateral lower extremity
289. (C) The spinal cord is organized into a series of tracts or neuropathways that carry motor (descending) and sensory (ascending) information. These tracts are organized anatomically within the spinal cord. The corticospinal tracts are descending motor pathways located anteriorly within the spinal cord. Axons extend from the cerebral cortex in the brain as far as the corresponding segment, where they form synapses with motor neurons in the anterior (ventral) horn. They decussate (cross over) in the medulla prior to entering the spinal cord. The dorsal columns are ascending sensory tracts that transmit light-touch, proprioception, and vibration information to the sensory cortex. They do not decussate until they reach the medulla. The lateral spinothalamic tracts transmit pain and temperature sensation. These tracts usually decussate within three segments of their origin as they ascend. The anterior spinothalamic tract transmits light touch. Autonomic function traverses within the anterior anteromedial tract. Sympathetic nervous system fibers exit the spinal cord between C7 and L1, while parasympathetic system pathways exit between S2 and S4. Injury to the corticospinal tract or dorsal columns, respectively, results in ipsilateral paralysis or loss of sensation of light touch, proprioception, and vibration. Unlike injuries of the other tracts, injury to the lateral spinothalamic tract causes contralateral loss of pain and temperature sensation two to three segments below the level of injury. Because the anterior spinothalamic tract also transmits light-touch information, injury to the dorsal columns may result in complete loss of vibration sensation and proprioception but only partial loss of light-touch sensation. Anterior cord injury causes paralysis and incomplete loss of light-touch sensation.
39
290. A38-year-old police officer reports continuous neck pain lasting past 6 months. The patient recalls lifting and carrying heavy boxes while moving his house and reports some neck pain at that time. Pain has gradually worsened over the past 6 months and now patient reports heaviness and occasional weakness in his right hand. The patient often feels numbness in right index finger as well. On examination, the patient has 5/5 strength in all muscle groups except mild weakness in flexors of the right elbow. Light-touch sensation is intact in all dermatomes, however, the patient reports increased sensation to light touch in the radial aspect of the right forearm. Deep tendon reflexes are intact bilaterally except for right brachioradialis reflex which is 1+ compared to left. This patient most likely has (A) right C5 radiculopathy (B) right C6 radiculopathy (C) right C7 radiculopathy (D) right C8 radiculopathy (E) cervical facet arthritis with referred pain
290. (B) Patients with a C6 radiculopathy should have pain in the neck, shoulder, lateral arm, radial forearm, dorsum of hand, and tips of thumb, index, and long finger. Distribution of pain is less extensive and more proximal, whereas paresthesias predominate distally. In some individuals, a C6 lesion will manifest as a depressed or absent biceps reflex; in others, an abnormal brachioradialis or wrist extensor reflex can be found. Elbow flexion will be weak, and the patient will be unable to supinate the forearm against resistance with the elbow held in extension. Conservative treatment includes physical therapy, traction, and analgesics. If pain persists, cervical epidural steroid injection may provide relief from pain and aid in physical therapy. However, if symptoms persist or weakness/numbness doesn’t improve surgical decompression with or without anterior fusion may be considered
40
291. A 42-year-old man underwent a celiac plexus block procedure with 20 mL of 50% alcohol. All of the following listed conditions are complications of this intervention EXCEPT (A) genitofemoral neuralgia (B) hypertension (C) diarrhea (D) paralysis (E) infection
291. (B) Celiac plexus block is both a diagnostic and therapeutic tool to help in managing upper abdominal pain arising from viscera. Pancreatic cancer is the leading diagnosis for neurolytic celiac plexus block; other conditions may include visceral pain arising from malignancies of liver or GI tract. The procedure is performed either under fluoroscopic guidance or CT scan, though blind approaches have also been described. Both single transaortic as well bilateral needle approaches have been described. The fluoroscopic image in question demonstrates a single needle transaortic celiac plexus block. Complications include diarrhea, hypotension, genitofemoral neuralgia, infection, bleeding, damage to surrounding structures and rarely paralysis. All complications mentioned above may occur except hypertension
41
``` 292. Atwo-needle lumbar sympathetic plexus block at L2 and L3 when performed appropriately may help in the diagnosis of (A) sympathetically mediated pain (B) lumbar discogenic pain (C) lumbar radiculopathy (D) diabetic neuropathy (E) facet arthritis ```
292. (A)Aproper diagnostic test requires a preblock patient evaluation (with special attention to the ipsilateral lower extremity pain, temperature, and condition), a local anesthetic injection using appropriate volume to avoid spread to adjacent nerves and a postblock evaluation of subjective improvement in pain score as well an objective increase in the temperature of the involved extremity is crucial. Significant improvement in pain scores with increase in temperature of the involved extremity points toward a positive diagnosis of sympathetically mediated pain. Discography is performed for diagnosis of lumbar discogenic pain. Whereas diabetic neuropathy may result in sympathetically mediated pain, it is a mixed somatic polyneuropathy and diagnosis is a clinical one. Lumbar selective nerve root block and facet joint injections may aid in the diagnosis of lumbar radiculopathy and facet arthritis resulting in pain.
42
293. A patient who received 1 cc of 0.25% bupivacaine after negative aspiration following a cervical selective nerve root injection became agitated and then developed generalized tonicclonic movements. Which of the following is the most likely explanation? (A) High spinal anesthetic from accidental intrathecal injection (B) Anxiety attack from pain during injection (C) Vertebral artery injection of local anesthetic (D) Injection into spinal cord (E) Hypoxia
293. (C) Cervical selective nerve root injection may be indicated for diagnosis and treatment of cervical radiculopathy. Complication other than infection, bleeding, and nerve damage, include intravascular uptake into vertebral artery or radicular arteries resulting in seizure, stroke, or paraplegia. Intraspinal spread into epidural or intrathecal spread is also possible resulting in high spinal anesthetic. Damage to spinal cord has also been reported with injection into the spinal cord. Considering the life-threatening complications, cervical selective nerve root block should only be performed by physicians well versed in this technique.
43
``` 294. Medial branch nerve blocks may aid in the diagnosis of (A) facet arthritis (B) sympathetically mediated pain (C) spinal nerve irritation (D) sciatica (E) myofascial pain ```
294. (A) Medial branches of the dorsal ramus provide innervations to the respective facet joint as well to the joint below. A diagnostic medial branch block with local anesthetic performed at appropriate levels (eg, L3 and L4 for L4-5 facet joint) may provide diagnostic and prognostic information to help with pain associated with facet arthritis. If pain is considerably albeit transiently improved after diagnostic medial branch blocks, a radiofrequency ablative procedure may be considered to provide longer lasting pain relief.
44
``` 295. Which of the following is the most likely side effect of a SI joint injection? (A) Perforation of bladder (B) Left lower extremity weakness (C) Stroke (D) High spinal resulting in cardiorespiratory depression (E) Injury to pudendal nerve ```
295. (B) SI joint injection is performed for both diagnostic and therapeutic reasons in patient complaining of SI joint pain. After a therapeutic injection with 5 to 10 mL of local anesthetic; it is possible that the local anesthetic may spill inferiorly and anteriorly and anesthetize sciatic nerve resulting in leg weakness. Patients may be warned about this, if observed afterward, and should be accompanied by a reasonable adult to avoid any falls and resultant injuries
45
``` 296. The potential complications of the vertebroplasty procedure include all EXCEPT (A) spinal cord compression (B) venous embolism (C) pedicle fracture (D) cement leak in soft tissue (E) bowl perforation ```
296. (E) Vertebroplasty is an advanced procedure that is performed to stabilize recently fractured vertebral bodies resulting in excruciating back pain. Performed properly and by trained physicians, vertebroplasty is a safe procedure. However, complications, though rare, are possible and uncompromising. These may include infection; bleeding; pulmonary embolus; damage to pedicles, spinal cord, or surrounding structures; allergic reactions to injectate; and cement leak into soft tissue or in spinal canal resulting in spinal cord compression.
46
297. A 70-year-old man reports severe cramps and “charley horse” sensation in both legs when walking more than one block. Resting usually helps in relieving pain. On examination patient reveals an intact neurologic examination without any sensory or motor deficit. Lower extremity examination reveals normal appearance, and no vascular insufficiency. Ankle brachial index performed 1 month ago is unremarkable. Which of the following is the most likely diagnosis? (A) Neurogenic claudication (B) Vascular claudication (C) Diabetic peripheral neuropathy (D) Amyloid neuropathy (E) Fibromyalgia
297. (A) Spinal stenosis may result from narrowing of the spinal canal secondary to hypertrophy of ligamentum flavum, articular processes and anteriorly from degenerative bulging discs. Stenosis may result in a classical presentation of neurogenic claudication with pain in lower legs or feelings of “charley horse” that come with walking an unpredictable distance and is relieved by resting or sitting down. In contrast to vascular claudication, the patient may report some back pain as well. In addition, pain is not predictably elicited after a certain walking distance because it is relative extension of the lumbar segments that results in worsening stenosis and neurogenic claudication rather than ischemia. Pain is relieved in neurogenic claudication by assuming a flexion posture (bending forward). Treatment includes posture education, education and improvement of body mechanics, and physical therapy. Epidural steroid injection series may provide pain relief in some patients. If pain or significant limitation in activity persists a decompressive laminectomy may be considered.
47
298. A32-year-old healthy female presents with a 2- month history of gradually worsening right lower extremity pain. The pain is sharp shooting in character and radiates down the right leg all the way to the right foot. On examination, patient has 5/5 muscle strength in all muscle groups except plantar flexors of the right ankle. The patient is unable to stand on her toes. There is no sensory deficit. Flexion, adduction, and internal rotation of the right hip results in reproduction of the symptoms. MRI of lumbar spine is normal with no evidence of herniated discs. This patient most likely has (A) right S1 radiculopathy (B) piriformis syndrome (C) SI arthritis (D) somatization disorder (E) discogenic pain
298. (B) The piriformis is a sausage-shaped muscle which originates from the anterior surface of the lateral sacrum and attaches to the greater trochanter. In most individuals the sciatic nerve lies anterior to the muscle belly. Spasm of the muscle may result in irritation of the sciatic nerve and resultant sciatica. The patient may report localized tenderness in the lower part of the buttock. In addition, if patients have irritation of sciatic nerve, they may also report symptoms suggestive of sciatica which may easily be confused with lumbar radiculopathy. However, flexion, adduction, and internal rotation of the thigh results in tightening of piriformis muscle which may reproduce pain symptoms. MRI should be carefully evaluated to rule out any radicular component. Treatments include muscle relaxants and physical therapy to break muscle spasm. If pain persists or if the patient is unable to continue with physical therapy, piriformis muscle injection may aid in treatment.
48
299. A 25-year-old construction worker, 8 months after a fall from a ladder, is unable to walk without assistance. However, worker compensations lawyers have provided video evidence of the patient being able to walk and also able to run with his dog. Which of the following is the most likely diagnosis? (A) Hypochondriasis (B) Factitious disorder (C) Malingering (D) Conversion disorder (E) Somatization disorder
299. (C) There can be physical and psychologic symptoms of malingering and factitious disorder. In these conditions the patient willfully produces or feigns symptoms of illness or injury. In the factitious disorder the goal of the behavior is the patient’s need to be in sick role—a need not understood by the patient. Placing blood into urine and pretending to have posttraumatic stress disorder are examples. There is no apparent external goal such as to obtain money or drugs. It is always a psychiatric illness. This contrast with malingering, in which there is a clearly defined external goal. Malingering is not a psychiatric illness. Diagnosis of hypochondriasis require atleast 6 month of preoccupation with the fear or belief that one has a serious disease, based on the interpretation of physical signs or sensations as evidence of illness. Somatization disorder is characterized by an extensive history of multiple somatic symptoms that are psychologic in nature. In addition to many physical complaints or a belief that one is sickly, the criteria require at least 13 symptoms from a list of 41. The symptom list includes 6 GI symptoms, 7 pain symptoms, 4 cardiopulmonary symptoms, 12 conversion/pseudoneurologic symptoms, 4 sexual symptoms, and 4 female reproductive symptoms. Conversion disorders are patients presenting with physical symptoms without any anatomic or pathophysiologic basis (pseudoneurologic symptoms; pseudoparalysis, pseudoseizure etc).
49
300. A 43-year-old gentleman has developed left groin pain 6 months after an inguinal hernia repair. The patient reports pain to be severe stabbing pain in the left groin radiating down to the left testicle. On examination, the patient has a well-healed incision and marked cutaneous allodynia and hyperalgesia. This patient most likely has (A) ilioinguinal neuralgia (B) mesh infection (C) recurrent hernia (D) wound dehiscence (E) incarceration
300. (A) Ilioinguinal neuralgia may develop after any surgery in inguinal area resulting in damage to the ilioinguinal nerve. Pain may start immediately after the surgery or may start after a reasonable period of healing has passed. Wound infection, recurrent hernia, and mesh infection should be ruled out to avoid any correctable causes of ilioinguinal pain. Pain is usually described as sharp, electrical sensation or sometimes as constant burning sensations in the groin area with occasional radiation into the testicles. Pain is exacerbated by light touch or rubbing of clothes. Treatment includes anticonvulsants and other adjuvant medications. If pain persists, local anesthetic diagnostic and therapeutic block as well as other treatment approaches may be warranted. Radiofrequency ablation, peripheral nerve stimulation, neurectomy, and repeat surgery should be reserved for resistant cases.
50
301. Which of the following is the most common complication from the celiac plexus block? (A) Hypotension (B) Seizure (C) Diarrhea (D) Hematoma (E) Subarachnoid injection
301. (A) A. Hypotension from sympathetic blockade is the most common complication. It is important to optimally prehydrate these patients prior to the onset of the block. B. Seizure results from intravascular injection of large volume of local anesthetic stressing the need to confirm negative aspiration prior to injecting the solution. C. Diarrhea ensues as a result of sympathetic blockade and unopposed parasympathetic tone. D. Retroperitoneal hematoma is a rare complication of celiac plexus block. E. Subarachnoid injection is the most serious and very rare complication celiac block.
51
302. A patient with history of three lumbar spinal fusions from an injury while working in a halfway home who is responsive to MS Contin (sustained-release morphine) 30 mg, three times a day and Norco (hydrocodone 5 mg with acetaminophen 325 mg) eight tablets per day with adequate analgesia and improved functionality, but limited activity secondary to side effects, receives an intrathecal opioid pump trial after been cleared by his psychologist. After confirmation of appropriate placement of the catheter under fluoroscopy, he is put on 0.5 mg/d of intrathecal morphine and gradually escalated up to 10 mg/d because of inadequate analgesia. Twelve hours after the procedure, he complains of nausea, headache, and sensation of “skin peeling off his body.” Which of the following is the best course of action in this case? (A) Increase the intrathecal morphine until pain relief and resolution of symptoms (B) CT scan of his spine to confirm correct placement of the catheter (C) Removal of the catheter and institution of oral opioids (D) Urine toxicology (E) Consultation with a spine surgeon
302. (D) A. Increase in the intrathecal morphine dose is warranted in some situations when a patient demonstrates signs and symptoms consistent with withdrawal or has inadequate analgesia. In that case it is important to carefully evaluate the equianalgesic dose accounting for change in route or incomplete cross tolerance with change of drugs. In this case, considering the oral to intrathecal conversion is 300 to 1, the patient has been escalated to 10 mg of intrathecal morphine a day; it seems unlikely that his symptoms would be because of opioid withdrawal provided his catheter is in the correct position as had been confirmed under fluoroscopy in this case. B. CT scan can be done to confirm the correct placement of the catheter if necessary; however it is highly unlikely that the catheter would move in a short time in a sedentary postsurgical patient. C. Removal of catheter followed by reimplantation is a possibility if indeed catheter is determined to be malpositioned. It seems rather premature to pursue such option at this time. D. Urine toxicology seems like a more viable option considering this patient’s association with a half way home and the time of onset of his symptoms approximately 12 hours after the hospitalization. Also, the symptoms experienced although nonspecific, point toward possible withdrawal from a substance of abuse. It is reasonable to order a urine/serum toxicology screen as an initial step at this point while instituting conservative treatment with nonopioid analgesics and antinausea preparations. E. Spine surgeon consultation does not seem necessary at this point since the symptoms experienced are not truly suggestive of spinal hematoma, infection, or neurologic deficits warranting acute surgical intervention.
52
``` 303. Migraine headaches are directly related to (A) estrogen increase (B) estrogen decrease (C) progesterone increase (D) progesterone decrease (E) none of the above ```
303. (B) The mechanism by which ovarian hormones influence migraines remain to be determined, but an abrupt decrease in serum estrogen concentrations before the onset of an attack appears to be a critical factor. Sometimes the use of percutaneous estrogen gel applied just before and through the menstrual cycle may reduce the frequency of headaches. However, in some other cases use of low-dose estrogen oral contraceptive formulation are associated with a haphazard occurrence of attacks during the cycle, probably because of fluctuating estrogen levels. Therefore, it seems prudent to have the treatment strategies aimed toward preventing either a decrease or substantial fluctuation in the levels of estrogen.
53
304. A50-year-old female comes in complaining of sudden onset pain in bilateral lower extremities and loss of bladder function. Her physical examination reveals motor weakness in her left lower extremity 3/5 compared to the right along with diminished sensation to light touch, pinprick, and temperature along L5 and S1 dermatomes on the right compared to the left. Rest of her physical, musculoskeletal, and neurologic examination is normal. Lumbosacral x-rays done by her primary care physician demonstrate anterolisthesis of L5 on S1. Which of the following is the most appropriate immediate action? (A) Consult the spine surgeon (B) Intravenous Opioids (C) Physical therapy (D) Reassurance and return to home with a follow-up visit in 2 weeks if symptoms persist (E) A course of oral steroids
304. (A) A. Considering the acute onset of bladder dysfunction and neurologic deficits on physical examination along with the anterolisthesis of L5 on S1, urgent evaluation by a spine surgeon seems to be the best immediate option of all. This patient needs further workup and possibly even urgent intervention by the spine surgeon at this time. B. While intravenous opioids can be used to control acute pain, they by no means should be considered adequate in managing this situation that demands urgent surgical attention. C. Physical therapy may be considered in future for this patient for physical rehabilitation once surgical evaluation and/or intervention has been completed. Physical therapy for acute pain management is inappropriate for this case considering the risk of neurologic deficits that may ensue from further movement of an unstable spine. D. This condition could be a surgical emergency and so this patient should be actively managed in an in-patient setting. E. Oral steroids may sometimes be beneficial in such setting to decrease the pain and inflammation associated with acute spine pain, but the surgical evaluation should take precedence over all conservative treatment options that may delay resolution of the spinal pathology.
54
``` 305. The approaches to celiac plexus block are all EXCEPT (A) retrocrural (B) transcrural (C) transaortic (D) intercrural (E) latera ```
305. (E) Celiac plexus or ganglia, these terms often used interchangeably, are a dense network of pre- and postganglionic fibers. The three splanchnic nerves; greater, lesser, and least synapse at the celiac ganglia. A. Retrocrural approach is the most commonly utilized by anesthesiologists and considered the most traditional. The landmarks include iliac crests, 12th rib, dorsal midline, vertebral bodies (T12-L2), and lateral border of the paraspinal (sacrospinalis) muscles. B. Transcrural approach involves placement of needle tips anterior and caudal to the diaphragmatic crura. Advocates of this approach believe that this approach maximizes spread of injected solutions anterior to the aorta where the celiac plexus is most concentrated and this minimizes the somatic nerve block. C. Transaortic approach to celiac plexus has been described under both fluoroscopic and CT guidance. It is considered safe by many because of the use of single fine needle compared to two-needle posterior approach. This approach has three distinct advantages over the classic two-needle approach. First, it avoids the risk of neurologic complications related to posterior retrocrural spread of drugs. Secondly, the aorta provides a definitive landmark for needle placement when radiographic guidance is not available and thirdly, much smaller volumes of local anesthetic and neurolytic solutions are required to achieve efficacy equal to or greater than that of classic retrocrural approach. D. Intercrural approach is a term that can technically be applied to transaortic approach since the needle tips are placed in front of the diaphragmatic crura in this approach, but more commonly this term is used to refer to the classic anterior approach to celiac plexus under CT or ultrasound guidance. E. Lateral approach has not been described in literature.
55
306. A 25-year-old male presents to you with leftsided neck pain with radiation along lateral aspect of the left arm, forearm, and thumb, index, and middle finger. He has associated tingling and numbness. On neurologic examination, the sensation to pinprick is diminished in the above mentioned distribution and brachioradialis jerk is lost on the left compared to intact 2+ on the right. The MRI of C-spine is compatible with an acute cervical disc herniation. Which of the following is the most appropriate initial treatment? (A) A course of oral opioids, oral steroids, and spine surgical consultation (B) A series of cervical epidural steroid injections under fluoroscopy (C) Physical therapy (D) Spinal cord stimulation (SCS) (E) Referral to pain psychologist for coping strategies
306. (A) A. The trial of oral opioids, steroids, and urgent consultation with a spine surgeon are the most appropriate initial steps in management of what seems to be a case of acute radiculopathy secondary to acute disc herniation. Because these substantial neurologic deficits may be reversed with appropriate and timely decompression the surgical evaluation and course of steroids are top priorities here. B. Cervical epidural steroid injections can be considered to decrease the inflammation, but does not qualify to be “most appropriate initial treatment.” C. Physical therapy can be instituted further down the road for rehabilitation. D. SCS may be beneficial to decrease neuropathic pain of chronic nature, but it has no role in an acute setting of this nature. E. Pain psychologist can prove to be very useful in patients suffering from chronic pain, but again has little role in acute pain management in this setting.
56
``` 307. Hoffmann sign is indicative of (A) upper motor neuron lesion (UMNL) (B) lower motor neuron lesion (LMNL) (C) radiculopathy (D) instability of cervical spine (E) malingering ```
307. (A) Hoffmann sign is indicative of UMNL. In fact, it is the upper extremity equivalent of Babinski reflex. The examiner holds the patient’s middle finger and briskly flicks the distal phalanx. A positive sign is noted if the interphalageal joint of thumb of the same hand flexes.
57
308. A 65-year-old male comes in complaining of pain in between the third and the fourth toes. The pain can be reproduced by palpation of the pulp between metatarsal heads. There is some relief of pain following localized administration of local anesthetic. Which of the following is the most likely diagnosis? (A) Plantar fascitis (B) Metatarsalgia (C) Tarsal tunnel syndrome (D) Morton neuroma (E) Painful calcaneal spur
308. (D) A. Plantar fascitis is an inflammation of the tendons and the fascia of the foot as they insert into the calcaneal periosteum. It is typically seen in the people who stand on hardwood floors for a prolonged period of time. Pain is elicited with plantar compression over the anterior calcaneus and also may radiate along plantar fascia. B. Metatarsalgia is characterized by pain in the plantar surface of the metatarsal heads caused by prolonged weight-bearing. It can also be replicated with manual compression over the metatarsal heads. Pain is most commonly increased in combined pronation and eversion. C. The etiology and diagnosis of tarsal tunnel syndrome is somewhat controversial. This syndrome involves compression or inflammation of the posterior tibial nerve that provides sensory innervation to medial aspect of the calcaneus, motor supply to small lateral musculature of the foot and to the medial and lateral plantar branches. The symptoms are usually vague with activity related problems. Pain along with paresthesia, cramping, and burning is seen in the distribution. Palpation reveals sensitivity in the area. EMG testing can be utilized in diagnosis of tarsal tunnel syndrome but is controversial. D. Morton neuroma (interdigital neuroma) is the compression of the interdigital nerves in between the metatarsal heads and deep transverse metatarsal ligaments. The third interspace between third and fourth metatarsal is most frequently involved, it is believed to be so because lateral plantar nerve sends a branch to the medial plantar nerve to form a larger third common digital nerve making it less mobile. The condition is usually unilateral and affects females more commonly than men, usually in their 50s. The most common symptom is plantar pain that is increased by walking or by palpation between the third and fourth metatarsal heads. E. Painful calcaneal spur is often seen in morbidly obese people or those who stand or walk excessively. Pain is increased in the morning or after a prolonged rest and similar to plantar fascitis except that it is more predominant in the posterior aspect of the plantar calcaneus.
58
``` 309. Which of the following is the most common nerve missed with the interscalene brachial plexus nerve block? (A) Ulnar (B) Radial (C) Musculocutaneous (D) Median (E) Axillary ```
309. (A) Interscalene block of brachial plexus is especially effective for surgery of the shoulder or upper arm, as the roots of the brachial plexus are most easily blocked with this technique. Ulnar nerve is most frequently spared since it is derived from the eighth cervical nerve and the block is placed at a more cephalic site with this approach. This block is ideal for reduction of a dislocated shoulder or any other type of surgery on shoulder or upper arm.
59
310. A 23-year-old gymnast while performing a double loop hears a popping sound in her left knee. Her knee immediately swells up and is very painful. On physical examination, tenderness on palpation and effusion is demonstrated. McMurray test is positive. Which of the following is the most likely diagnosis? (A) Baker cyst (B) Anterior cruciate ligament tear (C) Posterior cruciate ligament tear (D) Torn medial meniscus (E) Pes anserine bursitis
310. (D) A. A baker (popliteal) cyst represents ballooning of the synovium-lined joint capsule, usually on the posteromedial aspect of the knee. It is usually a secondary manifestation of underlying condition that causes chronic inflammation of the knee, such as meniscal tear, knee synovitis or intra-articular loose body. The diagnosis of the popliteal cyst can be made by direct palpation of the mass. Arthrography or an MRI can verify the diagnosis and demonstrate its communication with the joint cavity. The cyst usually resolves with correction of the underlying pathology. B. Anterior cruciate ligament is the most commonly injured knee ligament in athletes. Injury to this ligament will result in a bloody knee effusion that is very indicative of this particular kind of injury. Three tests used to diagnose anterior cruciate ligament injury are anterior drawer test, Lachman test, and pivot shift test. C. Posterior cruciate ligament is usually damaged in violent, usually high–kinetic energy injuries. These usually occur in combination with fractures, specifically to the patella and hip or with other knee ligament injuries. Injury to popliteal artery should be evaluated in this injury with palpation or even arteriography. The test used to diagnose posterior cruciate ligament injury is posterior drawer test. D. In stance more than 60% of the body’s weight is carried on the peripheral aspect of the tibial plateau by meniscal fibrocartilages. In younger persons, the meniscal injuries usually accompany other ligament injuries whereas in elderly, these usually occur in isolation. When a meniscal tear is extensive it can result in block to terminal knee flexion or extension, commonly described by patients as “locking of the knee”. A torn meniscus can cause knee swelling and pain as it irritates the joint surface or synovium. Chronic meniscal injuries can result in arthritic joint surface. Joint line tenderness is found in about 50% of these injuries. McMurray test is used to detect tear of the meniscus that can be displaced. It is performed by flexing and extending the knee between 90° and 140° of flexion. One of the examiner’s hands rotates the tibia at the ankle while the other hand is placed in front of the joint line. This is followed by the extension of knee in the rotated position. A palpable click indicates an unstable tear of the meniscus. The Apley grind test can help distinguish between tear in the anterior or posterior portion of the meniscus. MRI or arthroscopy can also be used as diagnostic tools to identify a meniscal lesion. E. Pes anserine bursa lies between the medial hamstring tendons (sartorius, gracilis, and semitendinosus) and proximal medial tibia. It is inferior to the joint line which helps distinguish from the medial joint line tenderness secondary to meniscal injury.
60
311. A 35-year-old female is rear ended at 45 mph resulting in acute neck pain that was diagnosed to be of musculoskeletal nature in the emergency room. On the next day, her symptoms progress to right upper extremity pain and weakness, both of which are exacerbated with ipsilateral flexion of her neck and reaching overhead. She has no neurologic deficits and MRI of her neck shows no obvious pathology. There is obliteration of the radial pulse with arm extension and abduction. Which of the following is the most likely diagnosis? (A) Brachial plexitis (B) Cervical degenerative disc disease (C) Whiplash injury (D) Pancoast tumor (E) Thoracic outlet syndrome
311. (E) A. Brachial plexitis is an acute disorder of that almost always begins with unilateral diffuse pain in the shoulder followed by weakness in the proximal muscles. Sensory disturbances are less pronounced than motor deficits. The pain usually subsides after the acute phase. Electrodiagnostic studies can help to establish the diagnosis. B. Cervical degenerative disc disease can result in diffuse axial pain in the neck or radicular pain along a particular dermatome corresponding to the nerve root involved if associated with a herniated nucleus pulposus. C. Whiplash injury typically follows a highimpact motor vehicle accident that results in axial neck pain. It has a musculoskeletal component to it and is frequently associated with facet joint involvement. D. Pancoast tumor is the tumor of the apex of the lung that typically involves the brachial plexus. Pain is a common presenting symptom usually involving the lower cervical nerve roots or trunks. CT scan or MRI can sometimes offer valuable diagnostic information. E. Thoracic outlet syndrome usually involves impingement of subclavian vessels and lower trunk of brachial plexus resulting in various degrees of vascular or neurologic compromise or both with local supraclavicular pain. The most common etiologies are cervical rib, hypertrophy of scalenus anticus, costoclavicular abnormalities, but nevertheless can result from an acute trauma. The pain and sensory changes are usually aggravated by any activity that extends the brachial plexus, including carrying heavy objects, abducting arms over the head or with repetitive movements of the arm. Motor weakness is seen in intrinsic muscles of the hand. The obliteration of radial pulse with arm extension or abduction or traction can be present and is called Adson or Allen test.
61
312. The following is true about the H reflex EXCEPT (A) in clinical practice H reflex is limited to calf muscles (B) it is recorded in gastrocnemius and soleus muscles by stimulating the posterior tibial nerve in the popliteal fossa (C) because of the distance the impulse travels, the latency of the H wave is shorter than the F wave (D) the H reflex recorded from the soleus muscle is primarily mediated by the S1 nerve root (E) H reflex is normal in L5 radiculopathy whereas is prolonged in S1 radiculopathy
312. (C) A. H wave responses, in adults can be obtained in lower extremities. H wave response is an electric equivalent of the ankle deep tendon reflex, when the tibial nerve is stimulated. B. The tibial nerve behind the knee in the popliteal fossa is stimulated and the impulse travels via afferent fibers to the spinal cord at the S1 level. After synapse in the cord, anterior horn cells produce a motor response that can be recorded in gastrocnemius and soleus muscles. C. H waves are true reflexes, F wave is not. Because the H wave has to travel to the level of cord in order to produce a response, the latency is longer compared to F wave. D. This is correct as explained in (B). E. Since the impulses are conducted through S1 nerve, H reflex is typically prolonged in S1 radiculopathy but may be normal in L5 radiculopathy.
62
``` 313. The arteria radicularis magna, also known as artery of Adamkiewicz arises from aorta, at the following spinal levels: (A) L4-5 (B) T9-12 (C) T5-8 (D) T11-12 (E) T5-9 ```
313. (B) The spinal cord receives its blood supply from three longitudinal arteries: a single anterior spinal artery and two posterior spinal arteries. The diameter of anterior spinal artery is greatest at the cervical and lower thoracic levels and narrowest at the midthoracic levels from T3-T9. This region of the cord is considered to be the “vulnerable zone” with respect to circulation. The anterior spinal artery is reinforced at a number of segmental levels by feeder arterial branches called anterior medullary feeder arteries. At the thoracic level, there are a total of eight of these feeder arteries, largest of which is called artery of Adamkiewicz or great anterior medullary artery. This artery typically enters the cord on the left side anywhere from T7 to L4, but most commonly at T9-T12.
63
314. A56-year-old male who is an avid golfer comes in with left elbow pain not relieved after antiinflammatory medication trial, warm compress, and physical therapy. He has not been able to play 18 holes recently and this is making him quite depressed. On examination, passive flexion or extension against resistance of his left wrist causes pain. Which of the following is the most probable diagnosis in this patient? (A) Posterior interosseous nerve entrapment (B) Medial epicondylitis (C) Lateral epicondylitis (D) de Quervain disease (E) Brachioradialis tendonitis
314. (C) A. The involvement of deep radial nerve is called posterior interosseous nerve entrapment. The symptoms are similar to radial tunnel syndrome including pain over the proximal dorsal forearm, with maximum tenderness at the site of radial tunnel that is 4 cm distal to the lateral epicondyle over the posterior interosseous nerve. The pain is typically elicited by attempting to resist extension of long finger. B. Medial epicondylitis or golfer’s elbow results in pain and exquisite tenderness over medial epicondyle that is further aggravated by flexion and pronation of the forearm and the wrist. C. Lateral epicondylitis or tennis elbow involves the extensor-supinator muscle mass, including extensor carpi radialis brevis, extensor digitorum communis, extensor carpi radialis longus, extensor carpi ulnaris, and supinator. The extensor carpi radialis is most commonly involved, mostly from repetitive movement of the wrist involving wrist flexion, elbow extension, and forearm pronation. Provocative test involves grasping or extending the wrist against resistance or supinating the forearm when sudden and severe pain is experienced in the area of lateral epicondyle. The patient’s being an “avid golfer” is a distractor here. D. de Quervain disease or tenosynovitis of the tendon sheath of extensor pollicis brevis and adductor pollicis longus causes swelling and tenderness of anatomic snuff box. E. Brachioradialis tendonitis results in pain in the lateral forearm, that is, region of brachioradialis tendon, the provocative tests described above typically do not elicit characteristic symptoms.
64
315. All of these cervical pathologies are seen in patients with rheumatoid arthritis EXCEPT (A) subaxial subluxation (B) cranial settling (C) posterior-longitudinal ligament thickening (D) atlantoaxial subluxation (E) instability of cervical-zygapophyseal joints
315. (C) Patients with cervical rheumatoid arthritis develop neck pain exacerbated by movement, with atlantoaxial disease producing pain in upper cervical spine and subaxial involvement producing pain in lower neck and clavicular areas. Neurologic involvement is seen in more advanced cases of spinal cord or nerve root compromise related to deformity and soft tissue hypertrophy. Plain radiography is useful in showing structural abnormalities and dynamic studies including flexion extension, oblique and open mouth frontal projections in identifying instability. Anterior subluxation of atlantoaxial joint is the most common form of cervical spine derangement followed by subaxial subluxation (between C3 and C7), lateral subluxation, cranial settling (vertical subluxation), and posterior subluxation. Also, the autoimmune inflammatory changes affect the synovium of zygapophyseal joints resulting in laxity and subsequent instability.
65
316. While undergoing lumbar sympathetic block for CRPS, patient complains of sudden onset of sharp ipsilateral groin and genital pain on injection of the contrast agent. Which of the following is the most likely cause of this symptom? (A) Trauma to L2 nerve root (B) Trauma to genitofemoral nerve (C) Psoas spasm (D) Epidural injection (E) Successful lumbar sympathetic block
316. (B) A. Trauma to L2 nerve root may cause ipsilateral groin pain, but is not the most likely cause. B. The most likely cause of the symptoms mentioned in the question is trauma to genitofemoral nerve. In fact, it is the most common complication associated with lumbar sympatholysis, particularly by the lateral approach. The incidence has been reported to be as high as 15%, but may be as low as 4% with a single-needle technique. Most cases are transient and resolve with conservative measures but others may last as long as 6 weeks. Repeat local anesthetic lumbar sympathetic block, TENS (transcutaneous electrical nerve stimulator) unit and intravenous lidocaine have all been described as options for remission of genitofemoral neuralgia. C. Psoas spasm is also sometimes seen but it typically produces discomfort in ipsilateral low back. (D) and (E) do not present as groin pain.
66
``` 317. Which of the following is the most common inherited neuropathy? (A) Familial amyloid polyneuropathy (B) Fabry disease (C) Porphyric neuropathy (D) Charcot-Marie-Tooth disease (E) Diabetic polyneuropathy ```
317. (D) Painful symptoms of Charcot-Marie-Tooth (CMT) disease have been described in the hypertrophic or demyelinating form (CMT-1). Pain may be described shooting, sharp, or burning in their toes, feet, ankles, and knees. Common presentation is in the first or second decade with difficulties walking or running.
67
``` 318. A 52-year-old man comes to your office complaining of 11/2 years of “burning” pain in the metatarsal areas of his left foot. Which of the following is the most likely diagnosis? (A) Posterior tibial neuritis (B) Plantar fasciitis (C) Morton neuroma (D) Tarsal tunnel syndrome (E) Hallux rigidus ```
318. (C) Morton neuroma may be considered in the spectrum of interdigital neuritis (compression neuropathy). It is usually between the third and fourth toes or less often between the fourth and fifth toes. The pain tends to be experienced more with walking and weight bearing while wearing shoes. The pain is generally alleviated with rest and removal of shoes. The pain may be reproduced by exerting pressure between the two toes implicated. Interdigital injection of local anesthetic relieves the pain.
68
319. In MRI of the lumbar spine T2-weighted images (1) are generally more time-consuming to obtain (2) are ideal to image the anatomic detail of end-plate reactive changes (3) exhibit increased sensitivity to higher water content and thus, may be useful in imaging infectious processes or inflammation (4) can be used in place of gadolinium- DTPA (diethylenetriamine pentaacetic acid) contrast in imaging of postoperative patients to differentiate scarring from intervertebral disc issues
319. (B) MRI, especially with T2-weighted images (though generally more time consuming to obtain) is useful in imaging conditions such as osteomyelitis, discitis, spinal cord compression, and malignancy. T1-weighted images provide reasonably good anatomic detail in imaging of end-plate reactive changes as well as postoperative scarring, but gadolinium-DTPA contrast should be used in postoperative patients to differentiate scarring from intervertebral discs.
69
320. In EMG and NCS, the H reflex (1) is the electrical equivalent of a muscle stretch reflex elicited by tendon tap (2) is mostly present in the soleus muscle but at times also can be elicited in the forearm flexor muscles (3) may be delayed or absent in S1 radiculopathy (4) latencies are length-dependent and should be adjusted for patient’s height
320. (E) The H reflex is examined utilizing a modified motor nerve conduction study technique. The H reflex is generally present in the soleus muscle and at time forearm flexor muscles. It may be more widespread in hyperreflexic conditions (eg, myelopathy) and pediatrics. Delayed or absence of the tibial H wave may reflect S1 radiculopathy or other neuropathic processes.
70
321. A previously healthy 83-year-old male presents to your office complaining of acute abdominal pain but without obvious etiology. Medical conditions which should be investigated include (1) pneumonia (2) inflammatory bowel disease (3) pyelonephritis (4) inferior wall myocardial infarction
321. (E) The elderly may seek medical attention for multiple problems with initial complaints of abdominal pain including: dissecting abdominal aortic aneurysm in diabetic ketoacidosis, pneumonia, pyelonephritis, inflammatory bowel disease, mesenteric ischemia, constipation, bowel obstruction, peritonitis, and druginduced GI mucosal irritation.
71
``` 322. Patients diagnosed with cubital tunnel syndrome may have (1) pain and numbness in the ulnar border of the forearm and hand (2) clawing of the small finge (3) Wartenberg sign (4) a deep aching sensation in the mid forearmr ```
322. (A) The ulnar nerve may be compressed in the cubital tunnel (cubital tunnel syndrome) which may lead to atrophy of the first dorsal interosseous muscle, clawing of the small finger, weakness of small finger adduction (Wartenberg sign) and eventually in chronic ulnar nerve compromise—with weakness of grip and pinch.
72
323. A 53-year-old male comes to your office complaining of foot pain (predominantly in the heel—but also with diffuse plantar symptoms) which also occurs at night and can be exacerbated by prolonged standing or walking. It is associated with weakness of the phalanges (impairing the pushing off phase of walking) as well as sensory loss and paresthesia. After a complete history and physical examination are completed, the differential diagnosis may include (1) plantar fasciitis (2) peripheral neuropathies (3) posterior tibial nerve entrapment (4) tarsal tunnel syndrome
323. (E) Tarsal tunnel syndrome is not a common source of foot discomfort and needs to be distinguished from multiple other causes of pain in the foot including: painful peripheral neuropathies, medial plantar nerve entrapment (which may occur in joggers), posterior tibial nerve entrapment symptoms tend to be located in medial plantar heel area, abductor digiti quinti nerve entrapment (usually with burning pain in heel pad area), and plantar fasciitis. Plantar fasciitis pain may be diffuse or migrate but with time is usually noted at the inferior aspect of the heel (around the medial calcaneal tuberosity) mainly, although typically severe with the first few steps in the morning, tends to diminish through the course of the day (unless intense or prolonged weight-bearing activity is under taken).
73
324. The diagnostic criteria for CRPS I—as accepted in 1994 by the International Association for the Study of Pain (IASP)—includes which of the following? (1) The presence of an initiating noxious event, or a cause of immobilization (2) Continuing pain, allodynia, or hyperalgesia with which the pain is disproportionate to any inciting event (3) Evidence at some time of edema, changes in skin blood flow, or abnormal sudomotor activity in the region of pain (4) This diagnosis is excluded by the existence of conditions that would otherwise account for the degree of pain and dysfunction
324. (E) Although, somewhat controversial and different from various proposed research criteria, the diagnosis of CRPS I, includes: 1. The presence of an initiating noxious event or a cause of immobilization. 2. Continuing pain, allodynia, or hyperalgesia with which the pain is disproportionate to any inciting event. 3. Evidence at some time of edema, changes in skin blood flow, or abnormal sudomotor activity in the region of the pain. 4. This diagnosis is excluded by the existence of condition that otherwise would account for the degree of pain and dysfunction.
74
325. The paroxysmal hemicranias are rare benign headache disorders that may typically be associated with (1) conjunctival injection (2) rhinorrhea (3) ptosis (4) eyelid edema
325. (E) Paroxysmal hemicranias may be chronic (CPH) (eg, daily) or episodic (EPH) (eg, discrete headache period or separated by periods of remission) characterized by severe, excruciating, throbbing, boring, or pulsatile pain affecting the orbital, supraorbital, and temporal regions. The pain tends to be associated with at least one of the following signs or symptoms ipsilateral to the painful side: 1. Conjunctival injection 2. Nasal congestion 3. Lacrimation 4. Ptosis 5. Rhinorrhea 6. Eyelid edema Attacks may occur at any time—occasionally waking patients from sound sleep and tend to last for 2 to 25 minutes (although may linger a couple of hours). The patient generally has 1 to 40 attacks per day.
75
``` 326. Which of the following statement(s) is (are) true? (1) The most common cause of thoracic radiculopathy is diabetes mellitus (2) The most common levels affected by herniated disc at cervical level are C4-5, C5-6, and C6-7 (3) L4-5 disc is more commonly herniates than L5-S1 (4) The nerve roots involved most commonly in thoracic outlet syndromes are C8 and T1 ```
326. (E) 1. Although thoracic radiculopathy has been described to result from multiple etiologies including tumor, scoliosis, infection, spondylosis, and herniated disc, diabetes mellitus is described as the most common cause. 2. The lower cervical discs are most commonly affected by herniation. 3. The frequency of L4-5 herniation is 45% compared to 42% at the level of L5-S1. With L4-5 herniation, L5 nerve root is most commonly affected. 4. Lower cervical nerve roots of brachial plexus, that is, C8 and T1 nerve roots are most commonly affected in thoracic outlet syndrome.
76
``` 327. The characteristics of conus medullaris syndrome include (1) asymmetric paraplegia (2) symmetric paraplegia (3) bladder function preservation (4) upper motor neuron lesion signs ```
327. (C) Epidural spinal cord compression is compression of spinal cord or cauda equina nerve roots from a lesion outside the dura mater. Epidural spinal cord or cauda equina compression is the second most common neurologic complication of cancer, occurring in up to 10% of patients. The most common tumors causing metastatic epidural compression are breast, lung, prostate, lymphoma, sarcoma, and kidney. Conus medullaris lesions typically cause a rapidly progressive symmetric perineal pain followed by early autonomic dysfunction, saddle sensory loss, and motor weakness. Limited straight leg raise test usually points to an epidural or intradural extramedullary lesion causing root compression, whereas segmental pain and sacral sparing suggest intramedullary disease.
77
328. Which of the following statement(s) is (are) true for central pain of spinal cord origin? (1) Most common etiology is of traumatic origin (2) Most common type of pain in these patients is spontaneous steady, burning, or dysesthetic pain affecting approximately 96% of patients (3) Bowel and bladder dysfunction can be seen in these patients (4) Most patients will develop cord central pain within 1 to 6 months of causative lesion although some may present more than 5 years out
328. (E) 1. The incidence of spinal cord pain has been estimated to be in the range of 6.4% to 94% of patients who experience spinal cord injury. 2. Patients may describe a variety of pain types; however, the three most common types are spontaneous steady, spontaneous neuralgic, and evoked pain including allodynia and hyperpathia. According to a study of 127 patients with spinal cord pain by Boureau and colleagues, 75% of patients reported burning pain. 3. Bowel and bladder dysfunction may be associated with spinal cord injury depending on the level and extent of injury. 4. Onset is typically within 1 to 6 months of the injury. When the onset was delayed beyond 1 year, 56% of the patients were found to suffer from a syrinx.
78
329. A positive Froment sign indicates which of the following? (1) Weakness of first dorsal interosseous (2) Weakness of flexor pollicis brevis (3) Weakness of adductor pollicis (4) Weakness of hypothenar muscles
329. (A) Froment sign is positive when ulnar nerve dysfunction is present. Froment sign is tested by placing a piece of paper between patient’s thumb and index finger and checking the position of the thumb as the examiner tries to pull the paper away from the patient. Normally the distal joint of the thumb remains in extension but if there is ulnar nerve dysfunction the tip of the thumb flexes significantly to increased pressure in attempt to keep the paper from moving.
79
``` 330. The potential for drug-induced painful neuropathies exist with which of the following agents? (1) Amiodarone (2) Metronidazole (3) Pyridoxine (4) Vincristine ```
330. (E) Drug-induced painful neuropathies may include toxoids (especially with doses greater than 200 mg/m2), cisplatinum, vincristine, amiodarone, metronidazole, and pyridoxine (especially at doses greater than 200-300 mg/d).
80
``` 331. Spinal cord stimulation (SCS) has been used for the treatment of (1) failed back surgery syndrome (2) CRPS (3) angina (4) peripheral vascular disease ```
331. (E) SCS has been utilized by clinicians for a variety of chronic pain issues. Although a large body of work has been published, precise mechanisms of action of SCS remain elusive. Animal studies suggest that SCS triggers release of serotonin, substance P, and γ-aminobutyric acid (GABA) within the spinal cord dorsal horn.